Line 9: Line 9:
 
By Jensen's Inequality, we get
 
By Jensen's Inequality, we get
  
<math>\phi(\int_Afd\lambda \leq \int_A\phi(f)d\lambda</math>
+
<math>\phi(\int_Afd\lambda) \leq \int_A\phi(f)d\lambda = \frac{\int_A\phi(f)d\mu}{\mu(A)}</math>

Revision as of 10:50, 22 July 2008

Define a function from the set of all measurable subset $ B $ of $ A $ as below

$ \lambda(B)=\frac{\mu(B)}{\mu(A)} $

This is clearly a measure on $ A $ with $ \lambda(A)=1 \frac{}{} $

Moreover, $ \int_{A}fd\mu = \mu(A)\int_A f d\lambda \frac{}{} $.

By Jensen's Inequality, we get

$ \phi(\int_Afd\lambda) \leq \int_A\phi(f)d\lambda = \frac{\int_A\phi(f)d\mu}{\mu(A)} $

Alumni Liaison

EISL lab graduate

Mu Qiao